Graph g(x)=f(x+1) when f(x) =4x-2

Answers

Answer 1

[tex]g(x)=4(x+1)-2[/tex]

[tex]g(x)=4x+4-2[/tex]

[tex]g(x)=4x+2[/tex]

Image attached below for graph.

Graph G(x)=f(x+1) When F(x) =4x-2

Related Questions

A digital camcorder repair service has set a goal not to exceed an average of 5 working days from the time the unit is brought in to the time repairs are completed. A random sample of 12 repair records showed the following repair times (in days): 5, 7, 4, 6, 7, 5, 5, 6, 4, 4, 7, 5.
H0: \mu \leq 5 days versus H1: \mu > 5 days. At \alpha = .05, choose the right option.
a) Reject H0 if tcalc < 1.7960
b) Reject H0 if tcalc >1.7960

Answers

Answer:

The degrees of freedom first given by:  

[tex]df=n-1=12-1=11[/tex]  

Then we can find the critical value taking in count the degrees of freedom and the alternative hypothesis and then we need to find a critical value who accumulates 0.05 of the area in the right tail and we got:

[tex] t_{\alpha}= 1.796[/tex]

And for this case the rejection region would be:

b) Reject H0 if tcalc >1.7960

Step-by-step explanation:

Information given

5, 7, 4, 6, 7, 5, 5, 6, 4, 4, 7, 5.

System of hypothesis

We want to test if the true mean is higher than 5, the system of hypothesis are :  

Null hypothesis:[tex]\mu \leq 5[/tex]  

Alternative hypothesis:[tex]\mu > 5[/tex]  

The statistic is given by:

[tex]t=\frac{\bar X-\mu_o}{\frac{s}{\sqrt{n}}}[/tex] (1)  

The degrees of freedom first given by:  

[tex]df=n-1=12-1=11[/tex]  

Then we can find the critical value taking in count the degrees of freedom and the alternative hypothesis and then we need to find a critical value who accumulates 0.05 of the area in the right tail and we got:

[tex] t_{\alpha}= 1.796[/tex]

And for this case the rejection region would be:

b) Reject H0 if tcalc >1.7960

Evaluate x - 2y when x = 5 and y = 5.
Determine whether the ordered pair satisfies the equation.
x - 2y = -5; (5,5)
Yes, the ordered pair satisfies the equation.
No, the ordered pair does not satisfy the equation.

Answers

Answer:

For the first question we just plug in the values so we get 5 - 2 * 5 = -5.

Again, for the second one we'll plug in the values and see if it's a true statement. 5 - 2 * 5 = -5 and -5 = -5 so the answer is yes.

At the kennel, the ratio of cats to dogs is 4:5. There are 27 animals in all. How many cats are in the kennel?

Answers

Answer:

Step-by-step explanation:

4x+5x=27

9x=27

x=27/9

x=3

4x3=12

5x3=15

The total number of cats were 12.

Based on the ratio of dogs to cats in the shelter, we know that out of 27 animals, there are 12 cats.

The ratio of cats to dogs is 4:5 which means that there are 5 dogs for every 4 cats.

This means that out of 9 animals, 4 would be cats and 5 would be dogs. If there was 27 animals therefore:

= 4 / 9 x 27

= 108 / 9

= 12 cats

In conclusion, there are 12 cats.

Find out more at https://brainly.com/question/9723361.

The mean height of women in a country​ (ages 20minus​29) is 64.2 inches. A random sample of 75 women in this age group is selected. What is the probability that the mean height for the sample is greater than 65 ​inches? Assume sigmaequals2.84. The probability that the mean height for the sample is greater than 65 inches is nothing.

Answers

Answer:

[tex] z=\frac{65-64.2}{\frac{2.84}{\sqrt{75}}} = 2.440[/tex]

And we can find the probability using the complement rule and with the normal standard table like this:

[tex] P(Z>2.440) =1-P(Z<2.440) = 1-0.993 =0.007[/tex]

The probability that the mean height for the sample is greater than 65 inches is 0.007

Step-by-step explanation:

Let X the random variable that represent the women heights of a population, and we know the following parameters

[tex]\mu=64.2[/tex] and [tex]\sigma=2.84[/tex]

We are interested on this probability

[tex]P(X>65)[/tex]

Since the sample size selected is 75>30 we can use the centrel limit theorem and the appropiate formula to use would be the z score given by:

[tex]z=\frac{x-\mu}{\frac{\sigma}{\sqrt{n}}}[/tex]

If we find the z score for 65 inches we got:

[tex] z=\frac{65-64.2}{\frac{2.84}{\sqrt{75}}} = 2.440[/tex]

And we can find the probability using the complement rule and with the normal standard table like this:

[tex] P(Z>2.440) =1-P(Z<2.440) = 1-0.993 =0.007[/tex]

The probability that the mean height for the sample is greater than 65 inches is 0.007

the time taken by a student to the university has been shown to be normally distributed with mean of 16 minutes and standard deviation of 2.1 minutes. He walks in once a day during term time, 180 days per year, and leaves home 20 minutes before his first lecture. a. Find the probability that he is late for his first lecture. b. Find the number of days per year he is likely to be late for his first lecture.

Answers

Answer:

a) 2.84% probability that he is late for his first lecture.

b) 5.112 days

Step-by-step explanation:

When the distribution is normal, we use the z-score formula.

In a set with mean [tex]\mu[/tex] and standard deviation [tex]\sigma[/tex], the zscore of a measure X is given by:

[tex]Z = \frac{X - \mu}{\sigma}[/tex]

The Z-score measures how many standard deviations the measure is from the mean. After finding the Z-score, we look at the z-score table and find the p-value associated with this z-score. This p-value is the probability that the value of the measure is smaller than X, that is, the percentile of X. Subtracting 1 by the pvalue, we get the probability that the value of the measure is greater than X.

In this question, we have that:

[tex]\mu = 16, \sigma = 2.1[/tex]

a. Find the probability that he is late for his first lecture.

This is the probability that he takes more than 20 minutes to walk, which is 1 subtracted by the pvalue of Z when X = 20. So

[tex]Z = \frac{X - \mu}{\sigma}[/tex]

[tex]Z = \frac{20 - 16}{2.1}[/tex]

[tex]Z = 1.905[/tex]

[tex]Z = 1.905[/tex] has a pvalue of 0.9716

1 - 0.9716 = 0.0284

2.84% probability that he is late for his first lecture.

b. Find the number of days per year he is likely to be late for his first lecture.

Each day, 2.84% probability that he is late for his first lecture.

Out of 180

0.0284*180 = 5.112 days

Jodie Meeks's Free Throws During the 2015-16 NBA season, Jodie Meeks of the Detroit Pistons had a free throw shooting percentage of 0.906 . Assume that the probability Jodie Meeks makes any given free throw is fixed at 0.906 , and that free throws are independent.If Jodie Meeks shoots 6 free throws in a game, what is the probability that he makes at least 5 of them?

Answers

Answer:

0.8973

Step-by-step explanation:

Relevant data provided in the question as per the question below:

Free throw shooting percentage = 0.906

Free throws = 6

At least = 5

Based on the above information, the probability is

Let us assume the X signifies the number of free throws

So,  Then X ≈ Bin (n = 6, p = 0.906)

[tex]P = (X = x) = $\sum\limits_{x}^6 (0.906)^x (1 - 0.906)^{6-x}, x = 0,1,2,3,.., 6[/tex]

Now

The Required probability = P(X ≥ 5) = P(X = 5) + P(X = 6)

[tex]= $\sum\limits_{5}^6 (0.906)^5 (1 - 0.906)^{6-5} + $\sum\limits_{6}^6 (0.906)^6 (1 - 0.906)^{6-6}[/tex]

= 0.8973

ASAP! GIVING BRAINLIEST! Please read the question THEN answer CORRECTLY! NO guessing. I say no guessing because people usually guess on my questions.

Answers

Answer: f(x)=2-x^2

Step-by-step explanation:

The quadratic equation is

y=ax^2+bx+c

and c is equal to the y-intercept.

in the twi graphs shown both have the same shape but different y-intervepts.

c(the y-intercept) in the first graph is 5 and in the second graph(F) is 2.

On the graphing calculator it says that f(x)=2-x^2 is the correct answer therefore it is correct.

The soup produced by a company has a salt level that is normally distributed with a mean of 5.4 grams and a standard deviation of 0.3 grams. The company takes readings of every 10th bar off the production line. The reading points are 5.8, 5.9, 4.9, 5.2, 5.0, 4.9, 6.2, 5.1, 5.7, 6.1. Is the process in control or out of control and why?

Answers

Answer:

Step-by-step explanation:

The mean of the reading points is

Mean = (5.8 + 5.9 + 4.9 + 5.2 + 5.0 + 4.9 + 6.2 + 5.1 + 5.7 + 6.1)/10 = 5.48

The process is out of control if the mean salt level of the readings is greater than 5.4

For the null hypothesis,

µ = 5.4

For the alternative hypothesis,

µ > 5.4

This is a right tailed test.

Since the population standard deviation is given, z score would be determined from the normal distribution table. The formula is

z = (x - µ)/(σ/√n)

Where

x = sample mean

µ = population mean

σ = population standard deviation

n = number of samples

From the information given,

µ = 5.4

x = 5.48

σ = 0.3

n = 10

z = (5.48 - 5.4)/(0.3/√10) = 0.84

Looking at the normal distribution table, the probability corresponding to the z score is 0.7996

The probability value to the right of the z score is 1 - 0.7996 = 0.2

Assuming a significance level of 0.05

Since alpha, 0.05 < than the p value, 0.2, then we would fail to reject the null hypothesis. Therefore, At a 5% level of significance, we can conclude that the process is not out of control. If we had rejected the null hypothesis, then our conclusion would be that the process is out of control.

What is the missing side length?

Answers

Answer:

8 yds

Step-by-step explanation:

The sides have to have the same length

14 yd = 6yd + ?

Subtract 6 from each side

14-6 = 8

8 yds

1/216^-2/3 + 1/256^-3/4 + 1/243^-1/5​

Answers

Answer:

103

Step-by-step explanation:

[tex]\dfrac{1}{216}^{-2/3}+\dfrac{1}{256}^{-3/4}+\dfrac{1}{243}^{-1/5}= \\\\\\\sqrt[3]{216^2}+\sqrt[4]{256^3}+\sqrt[5]{243}=\\\\\\6^2+4^3+3=\\\\\\36+64+3=\\\\\\103[/tex]

Hope this helps!

Please answer this correctly

Answers

Answer:

[tex]h=\sqrt{1.44}\\h = 1.2[/tex]

Step-by-step explanation:

Base of the triangle on the left = 0.5

Use pythagorean theorem

[tex]a^{2} + b^{2} = c^{2}[/tex]

Substitute

[tex]0.5^{2} + b^{2} = 1.3^{2}[/tex]

[tex]b^{2} = 1.3^2 - 0.5^2[/tex]

[tex]b^2 = 1.44[/tex]

[tex]b = \sqrt{1.44} \\[/tex]

[tex]b = 1.2[/tex]

in this case b is the height

so

[tex]h=\sqrt{1.44}\\h = 1.2[/tex]

What’s the correct answer for this question?

Answers

Answer:

D

Step-by-step explanation:

The volume of pyramid = 1/3 wlh

Where w = width, l = length and h = height

While,

The volume of rectangular prism = wlh

So,

The volume of pyramid = 1/3(the volume of prism)

Which number is irrational

Answers

Answer:

Can you give the question. Can you post the picture. I can help solve. I will edit this answer once you have given the question/picture.

Which ordered pair is the solution of the system of equations? 3x+2y=4, -2+2y=24

Answers

Answer:

x = -7.33        OR         x = [tex]\frac{-22}{3}[/tex]

y = 13

Step-by-step explanation:

→You can use the substitution method. First, make y by itself in (-2 + 2y = 24):

-2 + 2y = 24

2y = 26

y = 13

→Then, plug in 13 for y into the other equation:

3x + 2y = 4

3x + 2(13) = 4

3x + 26 = 4

3x = -22

x = -7.33        OR         x = [tex]\frac{-22}{3}[/tex]

80 81 82 83 84 85 86 87 88 89 90

Anika's test scores are shown below.

Anika's Test Scores

80 81 82 83 84 85 86 87 88 89 90

Which statement compares the shape of the two dot plots?

There is a gap in both plots.

There is a gap in Anika's scores, but not in Lorenzo's scores.

The data is widely spread across both plots.

The data is more widely spread for Lorenzo's scores than for Anika's.

Mark this and return

Save and Exit

Answers

Answer:

D :)

Step-by-step explanation:

Answer:

D

Step-by-step explanation:

Braily please

What is the difference of the polynomials? (–2x3y2 + 4x2y3 – 3xy4) – (6x4y – 5x2y3 – y5)

Answers

Answer:

-6x⁴y - 2x³y² + 9x²y³ - 3xy⁴ + y⁵

Step-by-step explanation:

(–2x³y² + 4x²y³ – 3xy⁴) – (6x⁴y – 5x²y³ – y⁵)=

–2x³y² + 4x²y³ – 3xy⁴ – 6x⁴y + 5x²y³ + y⁵=

-6x⁴y - 2x³y² + 9x²y³ - 3xy⁴ + y⁵

What is the volume of the rectangular prism?

Answers

Answer:

10ft[tex]{3}[/tex]

Step-by-step explanation:

One face has 15 blocks of 1/3 ft. You can clearly see 2 sets of blocks.

15 x 2 = 30

30 ÷ 3 or 30 x 1/3

= 10 ft cubed

In order to understand reasons why consumers visit their store, a local business conducts a survey by asking the next 100 people who visit their store to fill out a short survey. The business finds that 40 of the 100 people state that the main reason they visited the store was because the store is running a sale on coats that week. A confidence interval is constructed for the population proportion of consumers who would visit the store because of the coat sale. Which confidence interval would be the narrowest?

a. 90%
b. 99%
c. 95%
d. 85%

Answers

Answer:

d. 85%

Step-by-step explanation:

In a sample with a number n of people surveyed with a probability of a success of [tex]\pi[/tex], and a confidence level of [tex]1-\alpha[/tex], we have the following confidence interval of proportions.

[tex]\pi \pm z\sqrt{\frac{\pi(1-\pi)}{n}}[/tex]

In which

z is the zscore that has a pvalue of [tex]1 - \frac{\alpha}{2}[/tex].

The margin of error is:

[tex]M = z\sqrt{\frac{\pi(1-\pi)}{n}}[/tex]

The higher the confidence level, the higher the value of z, which means that the margin of error will be higher and the interval will be wider,

Which confidence interval would be the narrowest?

The one with the lowest confidence level. So the answer is d.

Write an equation of a line that passes through (-6, 1), parallel to y = 2x – 6.

Answers

Answer:

y = -1/2x - 2

Step-by-step explanation:

If it's parallel, that means that the slope is the opposite of the one in the given equation, meaning that 2 would be flipped and turned negative into -1/2.

Then, fill in the x and y values to get the y-intercept.

1 = -1/2(-6) + b

1 = 3 + b

-2 = b

So your answer is y = -1/2x - 2

Help asap giving branlist!!!

Answers

Answer:

D.

Step-by-step explanation:

So you know you have to have $62 as the base fee.

If you exceed 2 gigabytes, you subtract that by 2 because you want to find how many gigabytes you're going over. You then multiply it by 30 to find the cost.

You get C = 62 + 30(g - 2)

Answer:

anwser is d because it is write.

Step-by-step explanation:

Skyler is out shopping and sees that striped shirts are on sale for
$19.00 each, and plaid pants are on sale for $19.50 each. He
buys 8 shirts and 6 pairs of pants. What is the total of his
purchase?

The total was $_______​

Answers

Answer:

His total is $269

Step-by-step explanation:

8x19 = 152

6x19.50 = 117

152+117 = 269

In a group, 10 freshmen have mean GPA of 3.5; 20 sophomores have a mean GPA of 2.9; 25 juniors have a mean GPA of 3.2; and 15 seniors have a mean GPA of 3.4. What is the mean of the entire group

Answers

Answer:

[tex] T_1 = 10*3.5 = 35[/tex]

[tex] T_2 = 20*2.9 = 58[/tex]

[tex] T_3 = 25*3.2 = 80[/tex]

[tex] T_4 = 15*3.4 = 51[/tex]

[tex] \bar X= \frac{T_1 +T_2 +T_3 +T_4}{10+20+25+15}= \frac{35+58+80+51}{10+20+25+15} = 3.2[/tex]

Step-by-step explanation:

For this case we have the following info given:

[tex] n_1= 10 , \bar X_1 = 3.5[/tex] for freshmen

[tex] n_2= 20 , \bar X_2 = 2.9[/tex] for sophomores

[tex] n_3= 25 , \bar X_3 = 3.2[/tex] for juniors

[tex] n_4= 15 , \bar X_4 = 3.4[/tex] for seniors

For this case we can use the formula for the sample mean in order to find the total of each group:

[tex] \bar X =\frac{\sum_{i=1}^n X_i}{n}[/tex]

[tex]T= \sum_{i=1}^n X_i = n *\bar X[/tex]

And replacing we got:

[tex] T_1 = 10*3.5 = 35[/tex]

[tex] T_2 = 20*2.9 = 58[/tex]

[tex] T_3 = 25*3.2 = 80[/tex]

[tex] T_4 = 15*3.4 = 51[/tex]

And the grand mean would be given by:

[tex] \bar X= \frac{T_1 +T_2 +T_3 +T_4}{10+20+25+15}= \frac{35+58+80+51}{10+20+25+15} = 3.2[/tex]

In a particular region, for families with a combined income of $75,000 or more, 15% of these families have no children, 35% of the families have one child, 45% have two children, and 5% have three children. Use this information to construct the probability distribution for X, where x represents the number of children per family for this income group. Arrange x in increasing order and write the probabilities P(x) as decimals

Answers

Answer:

The probability distribution for x:"number of children per family for this income group" is:

[tex]\text{P(x=0)}=0.15\\\\\text{P(x=1)}=0.35\\\\\text{P(x=2)}=0.45\\\\\text{P(x=3)}=0.05\\\\[/tex]

Step-by-step explanation:

With the information given we have the relative frequencies of each category.

We know:

[tex]\text{P(x=0)}=0.15\\\\\text{P(x=1)}=0.35\\\\\text{P(x=2)}=0.45\\\\\text{P(x=3)}=0.05\\\\[/tex]

If 6 newborn babies are randomly selected, how many different gender sequences are possible?

Answers

Answer:

720

Step-by-step explanation:

6!

6x5x4x3x2x1=720

FIND P(NOT 6) WHEN YOU ROLL A STANDARD NUMBER CUBE THEN DESCRIBE THE LIKELIHOOD OF THE EVENT WRITE IMPOSSIBLE ,UNLIKELY , EQUALLY LIKELY , LIKLEY OR CERAIN

Answers

Answer: LIKLEY

Step-by-step explanation:

Formula : Probability [tex]=\dfrac{\text{Number of favorable outcomes}}{\text{Total outcomes}}[/tex]

A standard cube has six numbers on it (1,2,3,4,5 and 6).

P( NOT 6) =[tex]\dfrac{\text{Numbers that are not 6}}{\text{Total numbers}}[/tex]

[tex]=\dfrac{5}{6}=0.8333[/tex]

We know that when the probability of any event lies between 0.5 and 1then the event is said to be likely to happen.

Since , P(not 6)=0.8333 which lies between 0 and 0.5.

That means, it is likely to happen.

Note :

When probability of having A = 0 , we call A as uncertain event.

When probability of having A = 1 , we call A as certain event.

When probability of having A = 0.5 , we call A as equally likely event.

When probability of having A lies between 0 and 0.5 , we call A as unlikely event.

When probability of having A lies between 0.5 and 1 , we call A as likely event.

Ralph is 3 times as old as Sara. In 4 years, Ralph will be only tice as old as Sara will be then.
If x represents Sara's age now, which of the following expressions represents Ralph's age in four years?

A. 3x
B. 2x+4
C. 3x+4

Answers

Answer:

In 6 years, Ralph will be only twice as old as Sara

Step-by-step explanation:

Answer:

The answer is C, 3x+4

Step-by-step explanation:

The “in four years” part translates to +4. The 3x translates to 3 times his current age. Hope this helped :)

An appliance repairman charges $25 plus $40 per hour for house calls. Write the rule as an equation that relates hours worked x and his fee y.

Answers

To get the total fee, you need to multiply the hourly rate by number of hours worked and add that to the flat fee of $25.

The equation would be y = 40x + 25

Find the area of the triangle

Answers

Answer:

Step-by-step explanation:

The formula for the area of a triangle is base*height divided by 2. Remember this because itll be important for everything you do in math relating to geometry and calculus. Assuming you go that far

[tex]\frac{base*height}{2} =\frac{14*8}{2} =\frac{112}{2} = 56 units^2[/tex]

Answer:

A =56 units^2

Step-by-step explanation:

The area of a triangle is given by

A =1/2 bh where 14 is the base and 8 is the height

A = 1/2 (14)8

A =56 units^2

Researchers recorded that a certain bacteria population declined from 750,000 to 250 in 48 hours after the administration of medication. At this rate of decay, how many bacteria will there be in 8 hours?

Answers

Answer:

There will be 66 bacteria in 8 hours.

Step-by-step explanation:

The number of bacteria after t hours is given by the following formula.

[tex]P(t) = P(0)(1-r)^{t}[/tex]

In which P(0) is the initual number of bacteria and r is the decay rate.

Researchers recorded that a certain bacteria population declined from 750,000 to 250 in 48 hours after the administration of medication.

This means that [tex]P(0) = 750000, P(48) = 250[/tex]

We use this to find r. So

[tex]P(t) = P(0)(1-r)^{t}[/tex]

[tex]250 = 750000(1-r)^{48}[/tex]

[tex](1-r)^{48} = \frac{250}{750000}[/tex]

[tex]\sqrt[48]{(1-r)^{48}} = \sqrt[48]{\frac{250}{750000}}[/tex]

[tex]1-r = 0.84637[/tex]

So

[tex]P(t) = 750000(0.84637)^{t}[/tex]

How many bacteria will there be in 8 hours?

8 hours from now, in this context, is 8 + 48 = 56 hours. So this is P(56).

[tex]P(56) = 750000(0.84637)^{56} = 65.83[/tex]

Rounding to the nearest number

There will be 66 bacteria in 8 hours.

Answer:

197,488

Step-by-step explanation:

This problem requires two main steps. First, we must find the unknown rate, k. Then, we use that value of k to help us find the unknown number of bacteria.

Identify the variables in the formula.

AA0ktA=250=750,000=?=48hours=A0ekt

Substitute the values in the formula.

250=750,000ek⋅48

Solve for k. Divide each side by 750,000.

13,000=e48k

Take the natural log of each side.

ln13,000=lne48k

Use the power property.

ln13,000=48klne

Simplify.

ln13,000=48k

Divide each side by 48.

ln13,00048=k

Approximate the answer.

k≈−0.167

We use this rate of growth to predict the number of bacteria there will be in 8 hours.

AA0ktA=?=750,000=ln13,00048=8hours=A0ekt

Substitute in the values.

A=750,000eln13,00048⋅8

Evaluate.

A≈197,488.16

At this rate of decay, researchers can expect 197,488 bacteria.

Please answer this correctly I have to finish this today as this is my deadline

Answers

Answer:

r = 1.499619733762 m  There is no such thing a quarter radius!

C = 9.4223886775301 m

A = 7.065 m^2

Step-by-step explanation:

Calculate r and C | Given A

Given the area of a circle calculate the radius and circumference

r = √(A / π)

C = 2πr

Agenda:

r = radius

C = circumference

A = area

π = pi = 3.1415926535898

√ = square root

Other Questions
Hewitt and Patel are partners, sharing gains and losses equally. They decide to terminate their partnership. Prior to realization, their capital balances are $30,000 and $20,000, respectively. After all noncash assets are sold and all liabilities are paid, there is a cash balance of $38,000. a. What is the amount of a gain or loss on realization Solve: x - 1 < 3 help me plssss Find two consecutive whole numbers that square root 2 lies between. Simplify the expression (3 - 4i)(1 + 5i) - (2 - i). Show your work please! I have the answer: 21 + `12i but I need to show my work, thank you! If u subtract 1/2 from a number and multiply the result by 1/2, u get 1/8, what is the number. please help whoever answers correctly I will mark them brainiest, rate them, and thanks to them. What is the molarity of a solution prepared by dissolving 7.0 moles of NaCl solute in 585 ml of solution? After World War I, many countries money was worth much less than before the war and food and fuel prices rose dramatically. This phenomenon is called: John takes out a personal loan for $5,000 at an annual interest rate 7.5%, compounded monthly. He plans to pay off the loan (including interest) in 3 years. In the ABC, the height AN = 24 in, BN = 18 in, AC = 40 in. Find AB and BC.btw i need two answers for bc not just one Infants gaining control of their torso before gaining control of their arms and legs is known as the cephalocaudal trend.Please select the best answer from the choices providedTrue or False Which equation describes the locus of points 5 units from points (3, -4)?A) (x+3)^2 + (y-4)^2=5B) (x-3)^2 + (y+4)^2=5C) (x-3)^2 + (y+4)^2=25D) (x+3)^2 + (y-4)^2=25***YOU DON'T HAVE TO EXPLAIN*** Which best describes an accomplishment of the woman suffrage movement?A. Women gained the right to join the military in most Western nations.B. Women gained equal pay for equal work with men.C. Women were admitted to universities in Europe by 1890.D. Women gained the right to vote in Western nations. rational numbers are always sometimes never natural numbers Franklin Delano Roosevelt's plan for America to get out of the Great Depression was called New Freedom.True or False The graph shows the numbers of miles two cars travel.Car B travels ___ more miles than Car A each hour. Use your responses to the previous questions, as well as your economic knowledge and terminology, to compose two well-written paragraphs that respond to the following question: Be sure to use these concepts in your writing: human capital health care PPC GDP infrastructure Your paragraphs should include a topic sentence as well as introductory and concluding sentences. What is the potential impact on the South African communitys local economy from the game reserves investment in the clinic? countries of Africa A jar of quarters and dimes contains $20.00. There are 110 coins in all. How many of each are there? The discriminant is b2 4ac = (8)2 (4)(2)(10) = What is the greatest common factor of 90 xy2 and 12xy?6xyB.6xyC.12xyD.12xy?Reset